M8130 Algebraic topology, tutorial 02, 2017 2.3.2017 Exercise 1. Show that (Sm , ∗) ∧ (Sn , ∗) ∼= (Sm+n , ∗). Solution. Using exercises 3 and 6 from the previous tutorial, we have (Sm , ∗) ∧ (Sn , ∗) ∼= (Dm /Sm−1 ) ∧ (Dn /Sn−1 ) ∼= ∼= (Dm × Dn )/(Sm−1 × Dm ∪ Dn × Sn−1 ) ∼= ∼= (Im × In )/(∂Im × In ∪ ∂In × Im ) = = Im+n /(∂(Im+n )) ∼= Dm+n /∂Dm+n ∼= Sm+n . Exercise 2. Show that CPn is a CW-complex. Solution. Clearly CP0 is a point. Next, we have CPn = Cn+1 \ {0}/{v ∼ λv, λ ∈ C \ {0}} ∼= S2n+1 \ {0}/{v ∼ λv, |λ| = 1} ∼= ∼= {(w, 1 − |w2|) ∈ Cn+1 , w ∈ D2n }/{w ∼ λw for |w| = 1} ∼= ∼= (D2n ∪ S2n−1 )/{w ∼ λw for w ∈ S2n−1 } = D2n ∪f C. Now taking the canonical projection S2n−1 → CPn−1 ∼= S2n−1 / ∼ as the attaching map f yields a CW-complex with one cell in every even dimension till 2n and none in the odd ones. Exercise 3. From the lecture we know that A := {1 n , n ∈ N} ∪ {0} as a subspace of R is not a CW-complex. Show that X := I × {0} ∪ A × I is not a CW-complex either. Solution. Suppose that X is a CW-complex. Then it cannot contain cells of dimension ≥ 2, because it becomes disconnected after removing any point. In fact, the space obtained after removing any point (a, 0) with a ∈ A has more than two connected components (three for a > 0, to be exact), so these points cannot lie inside a 1-cell. Therefore these points must form 0-cells, but we already know that A does not have discrete topology, a contradiction. Exercise 4. Show that the Hawaiian earring given by X = {(x, y) ∈ R2 , (x − 1 n )2 + y2 = 1 n2 for some n} is not a CW-complex. Solution. Suppose that X is a CW-complex. Using similar arguments as in the previous exercise, we can see that (0, 0) must be a 0-cell and that X must have either innitely many 0-cells, or innitely many 1-cells. But since X is compact, exercise 5 implies that X can have only nitely many cells, a contradiction. Exercise 5. Prove that every compact set A in a CW-complex X can have a nonempty intersection with only nitely many cells. M8130 Algebraic topology, tutorial 02, 2017 2.3.2017 Solution. X is comprised of cells that are indexed by elemnts of some set J. Let B be a set containing exactly one point from each intersection A ∩ eβ , β ∈ J. We need to show that B is closed and discrete, which will imply that B is compact (since B ⊆ A) and discrete, hence nite. We know that a set C ⊆ Xn is closed i both C ∩ Xn−1 and C ∩ en α for each α ∈ J are closed, because Dn ∪f Xn−1 is a pushout. Using induction, this implies that C ⊆ X is closed i C ∩ eα is closed for each α ∈ J. Since B ∩ eα contains at most one point for any α ∈ J and X is T1 (even Hausdor), this shows that B is closed. Using the same argument, B with any one point removed is closed. Therefore B is also discrete and we are done. Exercise 6. Show that for a short exact sequence 0 → A f −→ B g −→ C → 0 of abelian groups (or more generally modules over a commutative ring) the following are equivalent: (1) There exists p : B → A such that pf = idA. (2) There exists q : C → B such that gq = idC. (3) There exist p : B → A and q : C → B such that fp + qg = idB. (Another equivalent condition is B ∼= A ⊕ C, with (p, g) and f + q being the respective inverse isomorphisms.) Solution. (1) =⇒ (2) and (3): Since g is surjective, for any c ∈ C there is some b ∈ B such that g(b) = c. Moreover, for any other b ∈ B such that also g(b ) = c, we have b − fp(b) = b − fp(b ), since b − b ∈ ker g = imf, so that b − b = f(a) and fp(b − b ) = fpf(a) = f(a) = b − b . This shows that we can correctly dene q(c) := b − fp(b) for any such b. Then we have gq(c) = g(b) − gfp(b) = g(b) = c (since gf = 0), which shows that gq = idC, and also qg(b) = b−fp(b), hence fp+qg = idB. (3) =⇒ (1) and (2): Applying f from the right to the equation fp + qg = idB yields fpf = f (since gf = 0), which together with the fact that f is injective implies pf = idA. Similarly, applying g from the left yields gqg = g, which together with the fact that g is surjective implies gq = idC. Exercise 7. Let 0 → A∗ f −→ B∗ g −→ C∗ → 0 be a short exact sequence of chain modules. We have dened the connecting homomorphism ∂∗ : Hn(C) → Hn−1(A) by the formula ∂∗[c] = [a], where ∂c = 0, f(a) = ∂b and g(b) = c. Show that this denition does not depend on a nor b. M8130 Algebraic topology, tutorial 02, 2017 2.3.2017 Solution. We have ∂a = 0 i f(∂a) = 0 (using injectivity of f) i 0 = ∂f(a) = ∂∂b, and the last condition is true. Now let b, b ∈ B be such that g(b) = g(b ) = c with a, a ∈ A such that f(a) = b, f(a ) = b . Then b − b ∈ ker g = imf, so b − b = f(a) for some a ∈ A. Therefore f(∂a) = ∂b − ∂b = f(a − a ) and the injectivity of f implies ∂a = a − a , hence [0] = [∂a ] = [a] − [a ] and we are done.